6
$\begingroup$

In my research it would be great if the following result is valid. In what follows, $\overline{\mathbb{Q}}$, $\overline{\mathbb{Q}}_n$ and $\overline{\mathbb{Q}}_{<n}$ denotes the set of algebraic numbers, $n$-degree algebraic numbers and algebraic numbers with degree at most $n-1$, respectively. Also $\mathcal{R}_{\mathcal{F}}$ denotes the set of zeroes of a family of polynomials $\mathcal{F}$.

Thus, my question is if the following two propositions are true (clearly, Prop. 2 implies in Prop. 1):

Prop 1. If $K_n:=\mathbb{Q}(\{\alpha: \alpha\in \overline{\mathbb{Q}}_{<n}\})$, then there exists an $n$-degree algebraic number $\gamma$ which does not belong to $K_n$.

Prop 2. Let $t<n$ be a positive integer and set $K_n:=\mathbb{Q}(\mathcal{R}_{\mathcal{F}})$, where $\mathcal{F}$ is the set of all $t$-degree polynomials in $\overline{\mathbb{Q}}_{<n}[x]$. Then there exists an $n$-degree algebraic number $\gamma$ which does not belong to $K_n$.

By using some results of the paper https://arxiv.org/pdf/0901.1335.pdf it is possible to prove the existence of the algebraic $\gamma\not\in K_n$ with arbitrarily large degree. However, I was not able to find any quantitative version of their result for degree exactly equals to $n$.

Any suggestions?

$\endgroup$
7
  • 2
    $\begingroup$ What if $t$ in Prop.2 is equal to $n$? Then $K_n$ would contain all $n$-degree algebraic numbers. $\endgroup$ Jun 16, 2022 at 4:44
  • $\begingroup$ It is easy to see that there are algebraic $\gamma\not\in K_n$ of arbitrarily large degree. Simply, take $\gamma$ be of degree $N$, where $N$ has a prime divisor at least $n$. $\endgroup$
    – GH from MO
    Jun 16, 2022 at 7:20
  • 2
    $\begingroup$ The set of algebraic numbers of degree exactly $n$ is not a field...It is not closed under addition or multiplication. $\endgroup$
    – GreginGre
    Jun 16, 2022 at 8:28
  • $\begingroup$ @GHfromMO Yes, of course but Jean wants degree exactly $n$, I believe, and though I thought it would follow easily, I could not find a convincing argument. $\endgroup$
    – Olivier
    Jun 16, 2022 at 8:51
  • 1
    $\begingroup$ @Olivier I reacted to the sentence "By using some results of the paper". My point was that the results of the quoted arXiv preprint are not needed. $\endgroup$
    – GH from MO
    Jun 16, 2022 at 9:29

2 Answers 2

5
$\begingroup$

Proposition 1 is true.

Let $x$ be an element of degree $n$ whose Galois group is isomorphic to $S_n$. Let $F$ be a finite subset of algebraic elements of degree $<n$. I claim that $x$ is not in the subfield generated by $F$.

Let $K$ be a finite Galois extension containing $F\cup\{x\}$. Let $G$ be its Galois group. Galois corresponds yields an order-reversing bijection between the set of subfields of $K$ and the set of subgroups of $G$. Under this bijection, for $z\in K$, the subfield $\mathbf{Q}(z)$ corresponds to a subgroup $H_z$ of $G$, of index $\deg(z)$. In particular, for $y\in F$, $H_y$ has index $<n$. The condition $\mathbf{Q}(x)\subset\mathbf{Q}(F)$ (which we assume by contradiction) means, since $\mathbf{Q}(F)$ is the smallest upper bound to $\{\mathbf{Q}(y):y\in F\}$. Hence it means that $H_x$ contains the intersection $\bigcap_{y\in F} H_y$. Let $F'$, resp. $X$ be the set of all conjugates of elements of $F$, resp of $x$ and $N=\bigcap_{y\in F'}H_y$, and $M=\bigcap_{x'\in X}H_{x'}$. So $N$ is a normal subgroup and $N\subset H_x$. Hence $N\subset M$. Since $N$ is an intersection of subgroups of index $<n$, $G/N$ embeds into $S_{n-1}^k$ for some $k$. Hence its quotient $G/M\simeq S_n$ is a subquotient of $S_{n-1}^k$. This is absurd.

[If $n\ge 5$ the simple group $A_n$ is not a subquotient of $S_{n-1}$, hence not of $S_{n-1}^k$. Also $S_n$ is not a subquotient of $S_{n-1}^k$ for $n=2,3,4$: for $n=2$ this is clear since $S_2\neq 1=S_1$; for $n=3$ $S_3$ has elements of order 3 but not $S_2^k$; for $n=4$ $S_4$ is not metabelian unlike $S_3^k$.]

$\endgroup$
2
$\begingroup$

Proposition 2 is false, although perhaps only for $n = 4$ (and $t = 2$ or $t = 3$). If $t = 2$, every algebraic number $\gamma$ of degree $4$ is contained in $K_{4}$.

Let $K$ be the Galois closure of $\mathbb{Q}(\gamma)$, and $G = {\rm Gal}(K/\mathbb{Q})$. If $G \simeq (\mathbb{Z}/2\mathbb{Z}) \times (\mathbb{Z}/2\mathbb{Z})$, $\mathbb{Z}/4\mathbb{Z}$, or the dihedral group of order $8$, then there is a quadratic field $L$ with $\mathbb{Q} \subseteq L \subseteq \mathbb{Q}(\gamma)$, and hence $\gamma$ is a root of a degree $2$ polynomial with coefficients in $L$ and $L \subseteq \overline{\mathbb{Q}}_{<4}$. This implies that $\gamma \in \mathcal{R}_{\mathcal{F}}$ and so $\gamma \in K_{4}$.

If $G \simeq A_{4}$, then is a degree $3$ extension $M/\mathbb{Q}$ that is Galois and contained in $K$. There are three quadratic extensions of $M$, say $M_{1}$, $M_{2}$ and $M_{3}$, that are contained in $K$. Each $M_{i}$ is a quadratic extension of $M$, and so if $M_{i} = \mathbb{Q}(\beta_{i})$, then $\beta_{i}$ is a root of a degree $2$ polynomial with coefficients in $M$, and hence a root of a degree $2$ polynomial with coefficients in $\overline{\mathbb{Q}}_{<4}$. This implies that $M_{i} \subseteq K_{4}$ and since $K = \langle M_{1}, M_{2}, M_{3} \rangle$, we have $K \subseteq K_{4}$.

If $G \simeq S_{4}$, the fact that $K/\mathbb{Q}$ and $K_{4}/\mathbb{Q}$ are both normal implies that $K \cap K_{4}$ is a normal extension of $\mathbb{Q}$. For an $S_{4}$ extension, there are only two normal subextensions: the discriminant field $\mathbb{Q}(\sqrt{D})$, and the splitting field of the resolvent cubic. However, there is a non-normal cubic extension $M$ (corresponding to a Sylow $2$-subgroup of $S_{4}$), and quadratic extensions $M_{1}$, $M_{2}$ and $M_{3}$ of $M$. As in the $A_{4}$ case, $M_{i} \subseteq K_{4}$. However, precisely one of the $M_{i}$ is the splitting field of the resolvent cubic, and so $\langle M_{1}, M_{2}, M_{3} \rangle \subseteq K_{4}$ has degree over $\mathbb{Q}$ greater than $6$. Since $K \cap K_{4}$ is normal and $K \cap K_{4} \supseteq \langle M_{1}, M_{2}, M_{3} \rangle$, we have $K \cap K_{4} = K$ and thus $K \subseteq K_{4}$.

$\endgroup$

Your Answer

By clicking “Post Your Answer”, you agree to our terms of service and acknowledge you have read our privacy policy.

Not the answer you're looking for? Browse other questions tagged or ask your own question.